470 lines
20 KiB
TeX
470 lines
20 KiB
TeX
\documentclass[a4paper,12pt, table]{/media/documents/Cours/Prof/Enseignements/Archive/2014-2015/tools/style/classExamen}
|
|
\usepackage{/media/documents/Cours/Prof/Enseignements/Archive/2014-2015/2014_2015}
|
|
\usepackage{multicol}
|
|
|
|
% Title Page
|
|
\titre{}
|
|
% \seconde \premiereS \PSTMG \TSTMG
|
|
\classe{\PSTMG}
|
|
\date{12 février 2015}
|
|
\duree{2h}
|
|
\typedoc{Devoir commun}
|
|
\ptpres{3}
|
|
|
|
\printanswers
|
|
|
|
\begin{document}
|
|
\titlepage
|
|
|
|
\begin{questions}
|
|
|
|
\question[6]
|
|
Une entreprise fabriquant des paniers décide d'arrêter progressivement sa production: de 2300 paniers en janvier, elle diminue chaque mois la production de 200 paniers.
|
|
|
|
On note $u_n$ le nombre de panier produit le n-ième mois.
|
|
|
|
\begin{parts}
|
|
\part Expliquer pourquoi la suite est arithmétique. Donner la raison.
|
|
\begin{solution}
|
|
La suite est arithmétique pour passer de la production d'un mois donné au mois suivant, on enlève 200. La raison est -200.
|
|
\end{solution}
|
|
\part Calculer $u_1$ et $u_2$.
|
|
\begin{solution}
|
|
$u_0$ est la production en janvier avant le début de la diminution, donc $u_0 = 2300$.
|
|
\begin{eqnarray*}
|
|
u_1 & = & u_0 - 200 = 2300 - 200 = 2100 \\
|
|
u_2 & = & u_1 - 200 = 2100 - 200 = 1900 \\
|
|
\end{eqnarray*}
|
|
|
|
\end{solution}
|
|
\part Donner l'expression explicite de la suite $u_n$.
|
|
\begin{solution}
|
|
Expression explicite de $u_n$
|
|
\begin{eqnarray*}
|
|
u_n & = & u_0 + n\times r = 2300 + n\times (-200) = 2300 - 200n
|
|
\end{eqnarray*}
|
|
\end{solution}
|
|
\part Calculer $u_{11}$.
|
|
\begin{solution}
|
|
Calcul de $u_{11}$, on utilise la formule explicite.
|
|
\begin{eqnarray*}
|
|
u_{11} & = & 2300 - 200\times 11 = 100
|
|
\end{eqnarray*}
|
|
\end{solution}
|
|
\part À partir de quel mois la production s'arrêtera-t-elle?
|
|
\begin{solution}
|
|
On a vu qu'au bout de 11 mois, la production serait de 100 paniers (c'est $u_{11}$). Donc le mois d'après, la production s'arrêtera. AU bout de 12 mois, la production se sera arretée.
|
|
\end{solution}
|
|
\end{parts}
|
|
|
|
\question[9]
|
|
Une étude dand un centre hospitalier donne les résultats suivants:
|
|
\begin{itemize}
|
|
\item 1600 personnes travaillent dans ce centre.
|
|
\item 30\% sont des hommes.
|
|
\item 55\% des infirmières sont des femmes.
|
|
\item 1\% du personnel total sont des hommes aides-soignants.
|
|
\item 18\% du personnel sont des médecins.
|
|
\item Il n'y a pas de femmes chirurgien.
|
|
\end{itemize}
|
|
\begin{center}
|
|
\begin{tabular}{|c|*{5}{c|}}
|
|
\hline
|
|
& Aides-soignants & Infirmiers & Médecins & Chirurgiens & TOTAL \\
|
|
\hline
|
|
Femmes & 784 & & & & \\
|
|
\hline
|
|
Hommes & & & & & \\
|
|
\hline
|
|
TOTAL & & 440 & & & \\
|
|
\hline
|
|
\end{tabular}
|
|
\end{center}
|
|
\begin{parts}
|
|
\part Reproduire le tableau et le completer avec les données qui ne nécéssites pas de calculs.
|
|
\begin{solution}
|
|
En vert.
|
|
\begin{center}
|
|
\begin{tabular}{|c|*{5}{c|}}
|
|
\hline
|
|
& Aides-soignants & Infirmiers & Médecins & Chirurgiens & TOTAL \\
|
|
\hline
|
|
Femmes & 784 & \Ovalbox{1} = 242 & \Ovalbox{2} = 94 & \cellcolor{green} 0& \Ovalbox{3} = 1120 \\
|
|
\hline
|
|
Hommes & \Ovalbox{4} = 16 & \Ovalbox{5} = 198 & \Ovalbox{6} = 194 & \Ovalbox{7} = 72 & \Ovalbox{8} = 480 \\
|
|
\hline
|
|
TOTAL & \Ovalbox{9} = 800 & 440 & \Ovalbox{10} = 288 & \Ovalbox{11} = 72 & \cellcolor{green} 1600\\
|
|
\hline
|
|
\end{tabular}
|
|
\end{center}
|
|
Les numéros seront utilisés dans la question suivante.
|
|
|
|
\end{solution}
|
|
\part Finir de completer le tableau en justfiant chaque calculs.
|
|
\begin{solution}
|
|
\begin{itemize}
|
|
\item \Ovalbox{8}: $\frac{1600\times30}{100} = 480$
|
|
\item \Ovalbox{1}: $\frac{440\times55}{100} = 242$
|
|
\item \Ovalbox{4}: $\frac{1600\times 1}{100} = 16$.
|
|
\item \Ovalbox{10}: $\frac{1600 \times 18}{100} = 288$
|
|
\item \Ovalbox{3}: $1600 - 480 = 1120$
|
|
\item \Ovalbox{9}: $784 + 16 = 800$
|
|
\item \Ovalbox{2}: $1120 - 784 - 242 = 94$
|
|
\item \Ovalbox{5}: $440 - 242 = 198$
|
|
\item \Ovalbox{6}: $288 - 94 = 194$
|
|
\item \Ovalbox{11}: $1600 - 800 - 440 - 288 -16= 72$
|
|
\item \Ovalbox{7}: $480 - 16 - 198 - 194 = 72$
|
|
|
|
\end{itemize}
|
|
\end{solution}
|
|
\part Quelle est la proportion de femmes aide-soignantes dans ce centre?
|
|
\begin{solution}
|
|
Proportion de femmes aide-soignantes dans le centre:
|
|
\begin{eqnarray*}
|
|
\frac{784}{1600} & = & 0,49 = 49\%
|
|
\end{eqnarray*}
|
|
|
|
\end{solution}
|
|
\part Quelle est la proportion de médecins parmi les femmes?
|
|
\begin{solution}
|
|
Proportion de médecins parmi les femmes
|
|
\begin{eqnarray*}
|
|
\frac{94}{1120} & = & 0,084 = 8,4\%
|
|
\end{eqnarray*}
|
|
|
|
\end{solution}
|
|
\part Quelle est la proportion de femmes parmi les médecins?
|
|
\begin{solution}
|
|
Proportion de femmes parmi les médecins
|
|
\begin{eqnarray*}
|
|
\frac{94}{288} & = & 0,326 = 32,6\%
|
|
\end{eqnarray*}
|
|
|
|
\end{solution}
|
|
\end{parts}
|
|
|
|
\pagebreak
|
|
\question[6]
|
|
Un article coûtait 250\euro au premier janvier 2004. Il a subi une inflation (augmentation) de 4,6\% en 2004 puis de 3,8\% en 2005.
|
|
\begin{parts}
|
|
\part Calculer sont prix au premier janvier 2005 (après la première augmentation) et au premier janvier 2006 (après la deuxième augmentation).
|
|
\begin{solution}
|
|
Prix en janvier 2005 (après une augmentation de 4,6\%)
|
|
\begin{eqnarray*}
|
|
250\times(1+\frac{4,6}{100}) & = & 261,5
|
|
\end{eqnarray*}
|
|
Prix en janvier 2006 (après l'augmentation de 3,8\%)
|
|
\begin{eqnarray*}
|
|
261,5\times(1 + \frac{3,8}{100}) & \approx & 271,4
|
|
\end{eqnarray*}
|
|
|
|
|
|
\end{solution}
|
|
\part Calculer la variation absolue entre le premier janvier 2004 et et le premier janvier 2006.
|
|
\begin{solution}
|
|
Variation absolue entre janvier 2004 et janvier 2006
|
|
\begin{eqnarray*}
|
|
y_2 - y_1 & = & 271,4 - 250 = 21,4
|
|
\end{eqnarray*}
|
|
\end{solution}
|
|
\part Calculer la variation relative entre le premier janvier 2004 et et le premier janvier 2006.
|
|
\begin{solution}
|
|
Variation relative entre janvier 2004 et janvier 2006
|
|
\begin{eqnarray*}
|
|
\frac{y_2 - y_1}{y_1} & = & \frac{271,4 - 250}{250} = 0,0856 = 8,56\%
|
|
\end{eqnarray*}
|
|
|
|
\end{solution}
|
|
\part Quelle inflation faudrait-il en 2006 pour que son prix atteigne 300\euro?
|
|
\begin{solution}
|
|
Si on veut atteindre 300\euro en 2006, il faudrai
|
|
\begin{eqnarray*}
|
|
\frac{y_2 - y_1}{y_1} & = & \frac{300 - 271,4}{271,4} = 0,105 = 10,5\%
|
|
\end{eqnarray*}
|
|
|
|
\end{solution}
|
|
\end{parts}
|
|
|
|
|
|
\question[10]
|
|
% Depuis repère 67 p 157
|
|
Un magasin a annoncé sa journée de promotion par une distribution de tracts sur lesquels était indiqué:
|
|
\begin{center}
|
|
\textit{Grande journée de promotion! Dépensez moins!}
|
|
\end{center}
|
|
|
|
\textbf{Partie 1}\\
|
|
Le tableau ci-dessous donne les montants en euros, arrondis à l'unité, des achats effectués par les 80 clients du magasin pendant une journée ordinaire.
|
|
|
|
\definecolor{lightgray}{gray}{0.9}
|
|
\rowcolors{1}{lightgray}{}
|
|
\begin{center}
|
|
\begin{tabular}{|*{8}{c|}}
|
|
\hline
|
|
2 &3 &5 &5 &5 &8 &8 &8\\
|
|
\hline
|
|
8 &10 &10 &10 &10 &10 &10 &10\\
|
|
\hline
|
|
11 &13 &14 &14 &14 &20 &20 &20\\
|
|
\hline
|
|
20 &20 &20 &21 &24 &24 &25 &26\\
|
|
\hline
|
|
30 &30 &30 &30 &30 &30 &31 &33\\
|
|
\hline
|
|
33 &35 &36 &38 &38 &38 &38 &38\\
|
|
\hline
|
|
39 &39 &39 &39 &39 &40 &40 &40\\
|
|
\hline
|
|
40 &40 &40 &40 &40 &40 &42 &42\\
|
|
\hline
|
|
42 &43 &43 &43 &44 &44 &45 &45\\
|
|
\hline
|
|
45 &45 &45 &46 &46 &47 &55 &60\\
|
|
\hline
|
|
\end{tabular}
|
|
\end{center}
|
|
\rowcolors{1}{}{}
|
|
|
|
\begin{parts}
|
|
%1pts
|
|
\part Tracer le tableau des effectifs de cette série statistique.
|
|
\begin{solution}
|
|
Tableau des effectifs
|
|
|
|
\begin{center}
|
|
\begin{tabular}{|r|*{29}{c|}}
|
|
\hline
|
|
Montants & 2 & 3 & 5 & 8 & 10 & 11 & 13 & 14 & 20 & 21 & 24 & 25 & 26 & 30 & 31 & 33 & 35 & 36 & 38 & 39 & 40 & 42 & 43 & 44 & 45 & 46 & 47 & 55 & 60 \\
|
|
\hline
|
|
Effectifs & 1 & 1 & 3 & 4 & 7 & 1 & 1 & 3 & 6 & 1 & 2 & 1 & 1 & 6 & 1 & 2 & 1 & 1 & 5 & 5 & 9 & 3 & 3 & 2 & 5 & 2 & 1 & 1 & 1 \\
|
|
\hline
|
|
\end{tabular}
|
|
\end{center}
|
|
|
|
\end{solution}
|
|
% J'aimerai qu'ils aient à tracer un histogramme ici!
|
|
\part
|
|
\begin{subparts}
|
|
%1pt
|
|
\subpart Déterminer le pourcentage de clients ayant effectué des achats pour un montant ne dépassant pas les 27\euro.
|
|
\begin{solution}
|
|
Pourcentage des clients ayant fait des achats pour moins de 27\euro
|
|
\begin{eqnarray*}
|
|
\frac{32}{80} & = & 0,4 = 40\%
|
|
\end{eqnarray*}
|
|
|
|
\end{solution}
|
|
%1pt
|
|
\subpart Déterminer le pourcentage de clients ayant effectué des achats entre 30\euro\; et 40\euro\; inclus.
|
|
\begin{solution}
|
|
Pourcentage des clients ayant fait des achats entre 30 et 40\euro.
|
|
\begin{eqnarray*}
|
|
\frac{30}{80} & = & 0,375 = 37,5\%
|
|
\end{eqnarray*}
|
|
\end{solution}
|
|
\end{subparts}
|
|
% 2pt
|
|
\part Calculer la moyenne de cette série statistique.
|
|
\begin{solution}
|
|
Calcul de la moyenne de cette série
|
|
\begin{eqnarray*}
|
|
\bar{x} & = & \frac{2 + 3 + 5\times 3 + 8 \times 4 + \cdots + 55 + 60}{80} = 29,29
|
|
\end{eqnarray*}
|
|
\end{solution}
|
|
\part
|
|
\begin{subparts}
|
|
%1pt
|
|
\subpart Déterminer le minimum et le maximum de cette série statistique.
|
|
\begin{solution}
|
|
En lisant le tableau de valeurs:
|
|
\begin{itemize}
|
|
\item Mininum: 2
|
|
\item Maximum: 60
|
|
\end{itemize}
|
|
\end{solution}
|
|
%2pts
|
|
\subpart Déterminer la médiane de cette série statistique.
|
|
\begin{solution}
|
|
Médiane de cette série. Dans le sujet les données sont déjà rangées par ordre croissant.
|
|
|
|
Effectif total: 80
|
|
|
|
Position de la médiane: $\frac{80}{2} = 40$ Donc la médiane se trouver entre la 40 et la 41ième valeur. Donc $Me =33 $.
|
|
\end{solution}
|
|
%2pts
|
|
\subpart Déterminer les quartiles de cette série statistique.
|
|
\begin{solution}
|
|
Position du premier quartile: $\frac{1}{4} \times 80 = 20$. Donc le premier quartile se trouve entre la 20ième et la 21ième valeur. Donc $Q_1 = 14$.
|
|
|
|
Position du troisième quartile: $\frac{3}{4} \times 80 = 60$. Donc le troisième quartile se trouve entre la 60ième et la 61ième valeur. Donc $Q_3 = 40$.
|
|
\end{solution}
|
|
\end{subparts}
|
|
|
|
\begin{EnvFullwidth}
|
|
|
|
\textbf{Partie 2}\\
|
|
Un étude similaire a été faite sur 80 clients lors d'une journée de promotion. Cette étude a donné le diagramme en boite suivant:
|
|
|
|
\begin{center}
|
|
\begin{tikzpicture}[xscale = 0.1]
|
|
\tkzInit[xmin=0,xmax=80,xstep=10]
|
|
\boxplot{1}{5}{45}{55}{63}{75}
|
|
\foreach \x in {0,10,...,90} \draw(\x,0)node[rotate=90] {$-$} node[below]{\x};
|
|
\draw[->] (0,0) -- (95,0);
|
|
\end{tikzpicture}
|
|
\end{center}
|
|
|
|
\end{EnvFullwidth}
|
|
%1pts
|
|
\part Quels sont les 5 informations que l'on peut lire sur ce diagramme à propos des dépenses des clients lors d'une journée de promotion?
|
|
\begin{solution}
|
|
Sur le diagramme en boite, on peut lire:
|
|
\begin{center}
|
|
Min = 5 \hfill $Q_1 = 45$ \hfill $Me = 55$, \hfill $Q_3 = 65$ \hfill Max = 75
|
|
\end{center}
|
|
\end{solution}
|
|
%1pts
|
|
\part Tracer l'un au dessus de l'autre les diagrammes en boites d'une journée de promotion et d'une journée ordinaire.
|
|
\begin{solution}
|
|
\begin{center}
|
|
\begin{tikzpicture}[xscale = 0.1]
|
|
\tkzInit[xmin=0,xmax=80,xstep=10]
|
|
\boxplot{1}{5}{45}{55}{63}{75}
|
|
\boxplot{3}{2}{14}{33}{40}{60}
|
|
\foreach \x in {0,10,...,90} \draw(\x,0)node[rotate=90] {$-$} node[below]{\x};
|
|
\draw[->] (0,0) -- (95,0);
|
|
\end{tikzpicture}
|
|
\end{center}
|
|
|
|
\end{solution}
|
|
%1pts
|
|
\part En comparant ces deux diagrammes en boite, commenter l'annonce du magasin.
|
|
\begin{solution}
|
|
On remarque que lors d'une journée ordinaire, 75\% des clients depensent moins de 40\euro alors que lors d'une journée de promotions, 75\% depensent plus de 45\euro. L'annonce du magasin donc fausse, les clients ne dépensent pas moins un jour de promotion.
|
|
\end{solution}
|
|
\end{parts}
|
|
|
|
\question[6]
|
|
\begin{itshape}
|
|
Cet exercice est un questionnaire à choix multiplies (QCM).
|
|
|
|
Pour chaque question, une seule des 4 réponses proposées est correcte.
|
|
|
|
Indiquer sur la copie le numéro de la question ainsi que la réponse choisie. Aucune justification n'est demandée.
|
|
|
|
Une réponse juste rapporte 1,5~point, une réponse fausse ou l'absence de réponse ne rapporte ni n'enlève de point. Si le total des points est négatif, la note attribuée à l'exercice est ramenée à 0.
|
|
|
|
\end{itshape}
|
|
\begin{parts}
|
|
|
|
\part Combien l'équation $-6x + x^2 + 9 = 0$ a-t-elle de solution?
|
|
\begin{multicols}{2}
|
|
\begin{subparts}
|
|
\subpart Aucune solution
|
|
\subpart Une solution
|
|
\subpart deux solutions
|
|
\subpart Une autre réponse
|
|
\end{subparts}
|
|
\end{multicols}
|
|
\begin{solution}
|
|
Pour connaître le nombre de solution d'une équation du 2nd degré, il faut calculer le discriminant ($\Delta$). Attention ici les coefficients n'étaient pas rangé comme il le faut. Ici $a = 1$, $b = -6$ et $c = 9$.
|
|
\begin{eqnarray*}
|
|
\Delta & = & b^2 - 4ac = (-6)^2 - 4\times 1 \times 9 = 36 - 36 = 0
|
|
\end{eqnarray*}
|
|
L'équation a donc une seule solution.
|
|
|
|
La bonne réponse est donc la (b)
|
|
\end{solution}
|
|
\part Quelle formule doit-t-on taper dans \texttt{B2}, puis recopié vers le bas pour completer le tableau
|
|
|
|
\includegraphics[scale=0.4]{./fig/tab_qcm}
|
|
|
|
\begin{subparts}
|
|
\subpart \Ovalbox{\texttt{=2*0*0-2*0+2}}
|
|
\subpart \Ovalbox{\texttt{=2*A1*A1-2*A1+2}}
|
|
\subpart \Ovalbox{\texttt{=2*B2*B2-2*B2+2}}
|
|
\subpart Une autre réponse
|
|
\end{subparts}
|
|
\begin{solution}
|
|
La première solution n'est pas valable car même si ce calcul donnerai le bon résultat pour la première valeur, on ne pourrai pas étirer la formule vers le bas.
|
|
|
|
La deuxième solution n'est pas non plus valable, car dans la case \texttt{A1} il y a la valeur \texttt{x} ce qui ne permet pas de calculer l'image de 0 par $f$.
|
|
|
|
La troisième solution ne marche pas non plus, car la case \texttt{B2} est la case où on veut que le résultat soit affiché.
|
|
|
|
La bonne formule aurait été \texttt{=2*A2*A2-2*A2+2}. Donc la bonne réponse est (d).
|
|
|
|
\end{solution}
|
|
|
|
\part Le tableau de signe de la fonction $f(x) = x^2 - 6x + 9$ est
|
|
\begin{subparts}
|
|
\subpart \begin{tikzpicture}[scale = 0.7]
|
|
\tkzTabInit[]{$x$/1,$f(x)$/1}{$-\infty$, $+\infty$}
|
|
\tkzTabLine{,+,}
|
|
\end{tikzpicture}
|
|
\subpart \begin{tikzpicture}[scale = 0.7]
|
|
\tkzTabInit[]{$x$/1,$f(x)$/1}{$-\infty$,3, $+\infty$}
|
|
\tkzTabLine{, +, z , +,}
|
|
\end{tikzpicture}
|
|
\subpart \begin{tikzpicture}[scale = 0.7]
|
|
\tkzTabInit[]{$x$/1,$f(x)$/1}{$-\infty$,-3, 3, $+\infty$}
|
|
\tkzTabLine{, +, z, -, z, +,}
|
|
\end{tikzpicture}
|
|
\subpart Une autre réponse
|
|
\end{subparts}
|
|
\begin{solution}
|
|
Pour tracer le tableau de signe d'un polynôme du 2nd degré, il faut commencer par calculer le discriminant (ce calcul a déjà été fait pour la première question du QCM)
|
|
\begin{eqnarray*}
|
|
\Delta & = & 0
|
|
\end{eqnarray*}
|
|
Il y a donc une seule racine.
|
|
\begin{eqnarray*}
|
|
x_1 & = & \frac{-b}{2a} = \frac{6}{2\times 1} = 3
|
|
\end{eqnarray*}
|
|
De plus ici $a = 1 > 0$ sont ce polynôme est toujours positif. La bonne réponse est donc la (b).
|
|
|
|
|
|
\end{solution}
|
|
\part Le graphique de la fonction $f(x) = -3x^2 + 3x + 1$ est
|
|
\begin{multicols}{2}
|
|
\begin{subparts}
|
|
\subpart \begin{tikzpicture}[baseline=(O.base),start chain, scale=0.5]
|
|
\repereNoGrid{-4}{4}{-4}{4}
|
|
\clip (-4,-4) rectangle (4,4);
|
|
\draw[very thick, domain=-4:4, color=red] plot [samples=100] (\x, {-3*\x*\x + 3*\x + 1});
|
|
\end{tikzpicture}
|
|
\subpart \begin{tikzpicture}[baseline=(O.base),start chain, scale=0.5]
|
|
\repereNoGrid{-4}{4}{-4}{4}
|
|
\clip (-4,-4) rectangle (4,4);
|
|
\draw[very thick, domain=-4:4, color=red] plot [samples=100] (\x, {3*\x*\x + 3*\x + 1});
|
|
\end{tikzpicture}
|
|
\subpart \begin{tikzpicture}[baseline=(O.base),start chain, scale=0.5]
|
|
\repereNoGrid{-4}{4}{-4}{4}
|
|
\clip (-4,-4) rectangle (4,4);
|
|
\draw[very thick, domain=-4:4, color=red] plot [samples=100] (\x, {3*\x + 1});
|
|
\end{tikzpicture}
|
|
\subpart Une autre réponse
|
|
\end{subparts}
|
|
|
|
\end{multicols}
|
|
\begin{solution}
|
|
On peut commencer par éliminer la réponse (c) car c'est la courbe représentative d'une fonction affine et là $f$ est un polynôme du 2nd degré.
|
|
|
|
Ici $a = -3 < 0$ donc les branches de la parabole doivent être vers le bas ce qui élimine la réponse (b).
|
|
|
|
On peut vérifier que la réponse (a) correspond à la fonction $f$. Pour cela on peut tracer le graphique de cette fonction sur la calculatrice ou faire l'étude du polynôme avec le discriminant.
|
|
\end{solution}
|
|
\end{parts}
|
|
|
|
|
|
\end{questions}
|
|
|
|
|
|
\end{document}
|
|
|
|
%%% Local Variables:
|
|
%%% mode: latex
|
|
%%% TeX-master: "master"
|
|
%%% End:
|
|
|